Let $K = F_2(x)$ and $q_t(x) = x^2-x-t in K[x]$. Show that $q_t$ is not solvable by radicals.












1












$begingroup$


Let $K = F_2(x)$ and $q_t(x) = x^2-x-t in K[x]$. Let $E$ be its splitting field. I need to prove that $Gal_{E/K} simeq mathbb{Z_2}$ and that $q_t$ is not solvable by radicals.



Any hint?










share|cite|improve this question









$endgroup$








  • 2




    $begingroup$
    Do you mean $K = mathbb{F}_2(t)$? In that case, the polynomial is irreducible over $K$ (why?) and separable (why?) so $E/K$ is indeed a Galois extension, and the degree of the extension must be $2$. The Fundamental Theorem of Galois theory implies that the Galois group is of the same order, so this should tell you what the Galois group is.
    $endgroup$
    – Tob Ernack
    Dec 4 '18 at 3:41






  • 1




    $begingroup$
    As for solvability by radicals, in principle the Galois group is solvable so you might think the extension $E/K$ is radical, but there is a catch because the characteristic of the base field is nonzero, and in fact is equal to the degree of the polynomial, so Galois' general criterion is not applicable.
    $endgroup$
    – Tob Ernack
    Dec 4 '18 at 3:45










  • $begingroup$
    A hint for a useful description of the Galois group: If $alpha$ is one of the zeros of $q_t(x)$ (in some extension field of $K$), then what is the other zero? Remember what you know about the sum of the zeros of a quadratic!
    $endgroup$
    – Jyrki Lahtonen
    Dec 4 '18 at 4:58
















1












$begingroup$


Let $K = F_2(x)$ and $q_t(x) = x^2-x-t in K[x]$. Let $E$ be its splitting field. I need to prove that $Gal_{E/K} simeq mathbb{Z_2}$ and that $q_t$ is not solvable by radicals.



Any hint?










share|cite|improve this question









$endgroup$








  • 2




    $begingroup$
    Do you mean $K = mathbb{F}_2(t)$? In that case, the polynomial is irreducible over $K$ (why?) and separable (why?) so $E/K$ is indeed a Galois extension, and the degree of the extension must be $2$. The Fundamental Theorem of Galois theory implies that the Galois group is of the same order, so this should tell you what the Galois group is.
    $endgroup$
    – Tob Ernack
    Dec 4 '18 at 3:41






  • 1




    $begingroup$
    As for solvability by radicals, in principle the Galois group is solvable so you might think the extension $E/K$ is radical, but there is a catch because the characteristic of the base field is nonzero, and in fact is equal to the degree of the polynomial, so Galois' general criterion is not applicable.
    $endgroup$
    – Tob Ernack
    Dec 4 '18 at 3:45










  • $begingroup$
    A hint for a useful description of the Galois group: If $alpha$ is one of the zeros of $q_t(x)$ (in some extension field of $K$), then what is the other zero? Remember what you know about the sum of the zeros of a quadratic!
    $endgroup$
    – Jyrki Lahtonen
    Dec 4 '18 at 4:58














1












1








1





$begingroup$


Let $K = F_2(x)$ and $q_t(x) = x^2-x-t in K[x]$. Let $E$ be its splitting field. I need to prove that $Gal_{E/K} simeq mathbb{Z_2}$ and that $q_t$ is not solvable by radicals.



Any hint?










share|cite|improve this question









$endgroup$




Let $K = F_2(x)$ and $q_t(x) = x^2-x-t in K[x]$. Let $E$ be its splitting field. I need to prove that $Gal_{E/K} simeq mathbb{Z_2}$ and that $q_t$ is not solvable by radicals.



Any hint?







galois-theory galois-extensions






share|cite|improve this question













share|cite|improve this question











share|cite|improve this question




share|cite|improve this question










asked Dec 4 '18 at 2:53









michaelabamichaelaba

627




627








  • 2




    $begingroup$
    Do you mean $K = mathbb{F}_2(t)$? In that case, the polynomial is irreducible over $K$ (why?) and separable (why?) so $E/K$ is indeed a Galois extension, and the degree of the extension must be $2$. The Fundamental Theorem of Galois theory implies that the Galois group is of the same order, so this should tell you what the Galois group is.
    $endgroup$
    – Tob Ernack
    Dec 4 '18 at 3:41






  • 1




    $begingroup$
    As for solvability by radicals, in principle the Galois group is solvable so you might think the extension $E/K$ is radical, but there is a catch because the characteristic of the base field is nonzero, and in fact is equal to the degree of the polynomial, so Galois' general criterion is not applicable.
    $endgroup$
    – Tob Ernack
    Dec 4 '18 at 3:45










  • $begingroup$
    A hint for a useful description of the Galois group: If $alpha$ is one of the zeros of $q_t(x)$ (in some extension field of $K$), then what is the other zero? Remember what you know about the sum of the zeros of a quadratic!
    $endgroup$
    – Jyrki Lahtonen
    Dec 4 '18 at 4:58














  • 2




    $begingroup$
    Do you mean $K = mathbb{F}_2(t)$? In that case, the polynomial is irreducible over $K$ (why?) and separable (why?) so $E/K$ is indeed a Galois extension, and the degree of the extension must be $2$. The Fundamental Theorem of Galois theory implies that the Galois group is of the same order, so this should tell you what the Galois group is.
    $endgroup$
    – Tob Ernack
    Dec 4 '18 at 3:41






  • 1




    $begingroup$
    As for solvability by radicals, in principle the Galois group is solvable so you might think the extension $E/K$ is radical, but there is a catch because the characteristic of the base field is nonzero, and in fact is equal to the degree of the polynomial, so Galois' general criterion is not applicable.
    $endgroup$
    – Tob Ernack
    Dec 4 '18 at 3:45










  • $begingroup$
    A hint for a useful description of the Galois group: If $alpha$ is one of the zeros of $q_t(x)$ (in some extension field of $K$), then what is the other zero? Remember what you know about the sum of the zeros of a quadratic!
    $endgroup$
    – Jyrki Lahtonen
    Dec 4 '18 at 4:58








2




2




$begingroup$
Do you mean $K = mathbb{F}_2(t)$? In that case, the polynomial is irreducible over $K$ (why?) and separable (why?) so $E/K$ is indeed a Galois extension, and the degree of the extension must be $2$. The Fundamental Theorem of Galois theory implies that the Galois group is of the same order, so this should tell you what the Galois group is.
$endgroup$
– Tob Ernack
Dec 4 '18 at 3:41




$begingroup$
Do you mean $K = mathbb{F}_2(t)$? In that case, the polynomial is irreducible over $K$ (why?) and separable (why?) so $E/K$ is indeed a Galois extension, and the degree of the extension must be $2$. The Fundamental Theorem of Galois theory implies that the Galois group is of the same order, so this should tell you what the Galois group is.
$endgroup$
– Tob Ernack
Dec 4 '18 at 3:41




1




1




$begingroup$
As for solvability by radicals, in principle the Galois group is solvable so you might think the extension $E/K$ is radical, but there is a catch because the characteristic of the base field is nonzero, and in fact is equal to the degree of the polynomial, so Galois' general criterion is not applicable.
$endgroup$
– Tob Ernack
Dec 4 '18 at 3:45




$begingroup$
As for solvability by radicals, in principle the Galois group is solvable so you might think the extension $E/K$ is radical, but there is a catch because the characteristic of the base field is nonzero, and in fact is equal to the degree of the polynomial, so Galois' general criterion is not applicable.
$endgroup$
– Tob Ernack
Dec 4 '18 at 3:45












$begingroup$
A hint for a useful description of the Galois group: If $alpha$ is one of the zeros of $q_t(x)$ (in some extension field of $K$), then what is the other zero? Remember what you know about the sum of the zeros of a quadratic!
$endgroup$
– Jyrki Lahtonen
Dec 4 '18 at 4:58




$begingroup$
A hint for a useful description of the Galois group: If $alpha$ is one of the zeros of $q_t(x)$ (in some extension field of $K$), then what is the other zero? Remember what you know about the sum of the zeros of a quadratic!
$endgroup$
– Jyrki Lahtonen
Dec 4 '18 at 4:58










1 Answer
1






active

oldest

votes


















2












$begingroup$

In general, $ X^p - X - t $ is never solvable by radicals over $ mathbb F_p(t) $ (for prime $ p $), and it always has Galois group $ C_p $.



To see that the Galois group is $ C_p $, let $ gamma $ be a root in a splitting field and note that the other roots are precisely $ gamma + k $ for $ k in mathbb F_p $, so the Galois group is generated by translations of the form $ gamma to gamma + k $ for $ k in mathbb F_p $. This means that there is no intermediate field between $ mathbb F_p(t) $ and the splitting field $ L $ of $ X^p - X - t $ (since by degree considerations it has no root in $ mathbb F_p(t) $) and we deduce that the polynomial is irreducible in $ mathbb F_p(t)[X] $.



To see that it is never solvable by radicals, it's easy to see this is implied by the following lemma through induction: (the algebraic closure is taken on purpose, and is applied only to $ mathbb F_p $)



Lemma. If $ K supset mathbb{bar{F}}_p(t) $ is a finite extension of $ mathbb {bar{F}}_p(t) $ over which $ X^p - X - t $ is irreducible, then for any prime $ q $, any $ y in K $ and any $ alpha notin K $, $ alpha^q in K $; $ X^p - X - t $ remains irreducible over $ K(alpha) $.



Proof. In the case $ q neq p $, note that by a general result in field theory, $ X^q - y $ is either irreducible in $ K[X] $ or it has a root in $ K $. Since $ K $ contains all roots of unity of order coprime to $ p $, this is equivalent to $ X^q - y $ completely splitting over $ K $, and thus either $ alpha in K $ or $ y $ has degree $ q $ over $ K $. We can see it's impossible for there to be a degree $ p $ subextension of $ K(y^{1/q})/K $ by a divisibility argument.



In the case $ q = p $, the extension $ K(alpha)/K $ is a purely inseparable extension, thus there is an equality of separability degrees $ [K(alpha): mathbb{bar{F}}_p(t)]_s = [K : mathbb{bar{F}}_p(t)]_s $. If $ K(alpha) $ contained a root of $ X^p - X - t $, then its separability degree over $ mathbb{bar{F}}_p(t) $ would be strictly greater than that of $ K $, so it follows that $ K(alpha) $ does not contain a root of $ X^p - X - t $, and by extension we conclude $ X^p - X - t $ is irreducible over $ K(alpha) $ by a similar Galois theory argument we used above.



Finally, notice that solvability in radicals over $ mathbb{bar{F}}_p(t) $ is equivalent to solvability in radicals over $ mathbb F_p(t) $, since the former is a radical extension of the latter (obtained by adjoining all roots of unity of order coprime to $ p $).






share|cite|improve this answer











$endgroup$













    Your Answer





    StackExchange.ifUsing("editor", function () {
    return StackExchange.using("mathjaxEditing", function () {
    StackExchange.MarkdownEditor.creationCallbacks.add(function (editor, postfix) {
    StackExchange.mathjaxEditing.prepareWmdForMathJax(editor, postfix, [["$", "$"], ["\\(","\\)"]]);
    });
    });
    }, "mathjax-editing");

    StackExchange.ready(function() {
    var channelOptions = {
    tags: "".split(" "),
    id: "69"
    };
    initTagRenderer("".split(" "), "".split(" "), channelOptions);

    StackExchange.using("externalEditor", function() {
    // Have to fire editor after snippets, if snippets enabled
    if (StackExchange.settings.snippets.snippetsEnabled) {
    StackExchange.using("snippets", function() {
    createEditor();
    });
    }
    else {
    createEditor();
    }
    });

    function createEditor() {
    StackExchange.prepareEditor({
    heartbeatType: 'answer',
    autoActivateHeartbeat: false,
    convertImagesToLinks: true,
    noModals: true,
    showLowRepImageUploadWarning: true,
    reputationToPostImages: 10,
    bindNavPrevention: true,
    postfix: "",
    imageUploader: {
    brandingHtml: "Powered by u003ca class="icon-imgur-white" href="https://imgur.com/"u003eu003c/au003e",
    contentPolicyHtml: "User contributions licensed under u003ca href="https://creativecommons.org/licenses/by-sa/3.0/"u003ecc by-sa 3.0 with attribution requiredu003c/au003e u003ca href="https://stackoverflow.com/legal/content-policy"u003e(content policy)u003c/au003e",
    allowUrls: true
    },
    noCode: true, onDemand: true,
    discardSelector: ".discard-answer"
    ,immediatelyShowMarkdownHelp:true
    });


    }
    });














    draft saved

    draft discarded


















    StackExchange.ready(
    function () {
    StackExchange.openid.initPostLogin('.new-post-login', 'https%3a%2f%2fmath.stackexchange.com%2fquestions%2f3025063%2flet-k-f-2x-and-q-tx-x2-x-t-in-kx-show-that-q-t-is-not-solvabl%23new-answer', 'question_page');
    }
    );

    Post as a guest















    Required, but never shown

























    1 Answer
    1






    active

    oldest

    votes








    1 Answer
    1






    active

    oldest

    votes









    active

    oldest

    votes






    active

    oldest

    votes









    2












    $begingroup$

    In general, $ X^p - X - t $ is never solvable by radicals over $ mathbb F_p(t) $ (for prime $ p $), and it always has Galois group $ C_p $.



    To see that the Galois group is $ C_p $, let $ gamma $ be a root in a splitting field and note that the other roots are precisely $ gamma + k $ for $ k in mathbb F_p $, so the Galois group is generated by translations of the form $ gamma to gamma + k $ for $ k in mathbb F_p $. This means that there is no intermediate field between $ mathbb F_p(t) $ and the splitting field $ L $ of $ X^p - X - t $ (since by degree considerations it has no root in $ mathbb F_p(t) $) and we deduce that the polynomial is irreducible in $ mathbb F_p(t)[X] $.



    To see that it is never solvable by radicals, it's easy to see this is implied by the following lemma through induction: (the algebraic closure is taken on purpose, and is applied only to $ mathbb F_p $)



    Lemma. If $ K supset mathbb{bar{F}}_p(t) $ is a finite extension of $ mathbb {bar{F}}_p(t) $ over which $ X^p - X - t $ is irreducible, then for any prime $ q $, any $ y in K $ and any $ alpha notin K $, $ alpha^q in K $; $ X^p - X - t $ remains irreducible over $ K(alpha) $.



    Proof. In the case $ q neq p $, note that by a general result in field theory, $ X^q - y $ is either irreducible in $ K[X] $ or it has a root in $ K $. Since $ K $ contains all roots of unity of order coprime to $ p $, this is equivalent to $ X^q - y $ completely splitting over $ K $, and thus either $ alpha in K $ or $ y $ has degree $ q $ over $ K $. We can see it's impossible for there to be a degree $ p $ subextension of $ K(y^{1/q})/K $ by a divisibility argument.



    In the case $ q = p $, the extension $ K(alpha)/K $ is a purely inseparable extension, thus there is an equality of separability degrees $ [K(alpha): mathbb{bar{F}}_p(t)]_s = [K : mathbb{bar{F}}_p(t)]_s $. If $ K(alpha) $ contained a root of $ X^p - X - t $, then its separability degree over $ mathbb{bar{F}}_p(t) $ would be strictly greater than that of $ K $, so it follows that $ K(alpha) $ does not contain a root of $ X^p - X - t $, and by extension we conclude $ X^p - X - t $ is irreducible over $ K(alpha) $ by a similar Galois theory argument we used above.



    Finally, notice that solvability in radicals over $ mathbb{bar{F}}_p(t) $ is equivalent to solvability in radicals over $ mathbb F_p(t) $, since the former is a radical extension of the latter (obtained by adjoining all roots of unity of order coprime to $ p $).






    share|cite|improve this answer











    $endgroup$


















      2












      $begingroup$

      In general, $ X^p - X - t $ is never solvable by radicals over $ mathbb F_p(t) $ (for prime $ p $), and it always has Galois group $ C_p $.



      To see that the Galois group is $ C_p $, let $ gamma $ be a root in a splitting field and note that the other roots are precisely $ gamma + k $ for $ k in mathbb F_p $, so the Galois group is generated by translations of the form $ gamma to gamma + k $ for $ k in mathbb F_p $. This means that there is no intermediate field between $ mathbb F_p(t) $ and the splitting field $ L $ of $ X^p - X - t $ (since by degree considerations it has no root in $ mathbb F_p(t) $) and we deduce that the polynomial is irreducible in $ mathbb F_p(t)[X] $.



      To see that it is never solvable by radicals, it's easy to see this is implied by the following lemma through induction: (the algebraic closure is taken on purpose, and is applied only to $ mathbb F_p $)



      Lemma. If $ K supset mathbb{bar{F}}_p(t) $ is a finite extension of $ mathbb {bar{F}}_p(t) $ over which $ X^p - X - t $ is irreducible, then for any prime $ q $, any $ y in K $ and any $ alpha notin K $, $ alpha^q in K $; $ X^p - X - t $ remains irreducible over $ K(alpha) $.



      Proof. In the case $ q neq p $, note that by a general result in field theory, $ X^q - y $ is either irreducible in $ K[X] $ or it has a root in $ K $. Since $ K $ contains all roots of unity of order coprime to $ p $, this is equivalent to $ X^q - y $ completely splitting over $ K $, and thus either $ alpha in K $ or $ y $ has degree $ q $ over $ K $. We can see it's impossible for there to be a degree $ p $ subextension of $ K(y^{1/q})/K $ by a divisibility argument.



      In the case $ q = p $, the extension $ K(alpha)/K $ is a purely inseparable extension, thus there is an equality of separability degrees $ [K(alpha): mathbb{bar{F}}_p(t)]_s = [K : mathbb{bar{F}}_p(t)]_s $. If $ K(alpha) $ contained a root of $ X^p - X - t $, then its separability degree over $ mathbb{bar{F}}_p(t) $ would be strictly greater than that of $ K $, so it follows that $ K(alpha) $ does not contain a root of $ X^p - X - t $, and by extension we conclude $ X^p - X - t $ is irreducible over $ K(alpha) $ by a similar Galois theory argument we used above.



      Finally, notice that solvability in radicals over $ mathbb{bar{F}}_p(t) $ is equivalent to solvability in radicals over $ mathbb F_p(t) $, since the former is a radical extension of the latter (obtained by adjoining all roots of unity of order coprime to $ p $).






      share|cite|improve this answer











      $endgroup$
















        2












        2








        2





        $begingroup$

        In general, $ X^p - X - t $ is never solvable by radicals over $ mathbb F_p(t) $ (for prime $ p $), and it always has Galois group $ C_p $.



        To see that the Galois group is $ C_p $, let $ gamma $ be a root in a splitting field and note that the other roots are precisely $ gamma + k $ for $ k in mathbb F_p $, so the Galois group is generated by translations of the form $ gamma to gamma + k $ for $ k in mathbb F_p $. This means that there is no intermediate field between $ mathbb F_p(t) $ and the splitting field $ L $ of $ X^p - X - t $ (since by degree considerations it has no root in $ mathbb F_p(t) $) and we deduce that the polynomial is irreducible in $ mathbb F_p(t)[X] $.



        To see that it is never solvable by radicals, it's easy to see this is implied by the following lemma through induction: (the algebraic closure is taken on purpose, and is applied only to $ mathbb F_p $)



        Lemma. If $ K supset mathbb{bar{F}}_p(t) $ is a finite extension of $ mathbb {bar{F}}_p(t) $ over which $ X^p - X - t $ is irreducible, then for any prime $ q $, any $ y in K $ and any $ alpha notin K $, $ alpha^q in K $; $ X^p - X - t $ remains irreducible over $ K(alpha) $.



        Proof. In the case $ q neq p $, note that by a general result in field theory, $ X^q - y $ is either irreducible in $ K[X] $ or it has a root in $ K $. Since $ K $ contains all roots of unity of order coprime to $ p $, this is equivalent to $ X^q - y $ completely splitting over $ K $, and thus either $ alpha in K $ or $ y $ has degree $ q $ over $ K $. We can see it's impossible for there to be a degree $ p $ subextension of $ K(y^{1/q})/K $ by a divisibility argument.



        In the case $ q = p $, the extension $ K(alpha)/K $ is a purely inseparable extension, thus there is an equality of separability degrees $ [K(alpha): mathbb{bar{F}}_p(t)]_s = [K : mathbb{bar{F}}_p(t)]_s $. If $ K(alpha) $ contained a root of $ X^p - X - t $, then its separability degree over $ mathbb{bar{F}}_p(t) $ would be strictly greater than that of $ K $, so it follows that $ K(alpha) $ does not contain a root of $ X^p - X - t $, and by extension we conclude $ X^p - X - t $ is irreducible over $ K(alpha) $ by a similar Galois theory argument we used above.



        Finally, notice that solvability in radicals over $ mathbb{bar{F}}_p(t) $ is equivalent to solvability in radicals over $ mathbb F_p(t) $, since the former is a radical extension of the latter (obtained by adjoining all roots of unity of order coprime to $ p $).






        share|cite|improve this answer











        $endgroup$



        In general, $ X^p - X - t $ is never solvable by radicals over $ mathbb F_p(t) $ (for prime $ p $), and it always has Galois group $ C_p $.



        To see that the Galois group is $ C_p $, let $ gamma $ be a root in a splitting field and note that the other roots are precisely $ gamma + k $ for $ k in mathbb F_p $, so the Galois group is generated by translations of the form $ gamma to gamma + k $ for $ k in mathbb F_p $. This means that there is no intermediate field between $ mathbb F_p(t) $ and the splitting field $ L $ of $ X^p - X - t $ (since by degree considerations it has no root in $ mathbb F_p(t) $) and we deduce that the polynomial is irreducible in $ mathbb F_p(t)[X] $.



        To see that it is never solvable by radicals, it's easy to see this is implied by the following lemma through induction: (the algebraic closure is taken on purpose, and is applied only to $ mathbb F_p $)



        Lemma. If $ K supset mathbb{bar{F}}_p(t) $ is a finite extension of $ mathbb {bar{F}}_p(t) $ over which $ X^p - X - t $ is irreducible, then for any prime $ q $, any $ y in K $ and any $ alpha notin K $, $ alpha^q in K $; $ X^p - X - t $ remains irreducible over $ K(alpha) $.



        Proof. In the case $ q neq p $, note that by a general result in field theory, $ X^q - y $ is either irreducible in $ K[X] $ or it has a root in $ K $. Since $ K $ contains all roots of unity of order coprime to $ p $, this is equivalent to $ X^q - y $ completely splitting over $ K $, and thus either $ alpha in K $ or $ y $ has degree $ q $ over $ K $. We can see it's impossible for there to be a degree $ p $ subextension of $ K(y^{1/q})/K $ by a divisibility argument.



        In the case $ q = p $, the extension $ K(alpha)/K $ is a purely inseparable extension, thus there is an equality of separability degrees $ [K(alpha): mathbb{bar{F}}_p(t)]_s = [K : mathbb{bar{F}}_p(t)]_s $. If $ K(alpha) $ contained a root of $ X^p - X - t $, then its separability degree over $ mathbb{bar{F}}_p(t) $ would be strictly greater than that of $ K $, so it follows that $ K(alpha) $ does not contain a root of $ X^p - X - t $, and by extension we conclude $ X^p - X - t $ is irreducible over $ K(alpha) $ by a similar Galois theory argument we used above.



        Finally, notice that solvability in radicals over $ mathbb{bar{F}}_p(t) $ is equivalent to solvability in radicals over $ mathbb F_p(t) $, since the former is a radical extension of the latter (obtained by adjoining all roots of unity of order coprime to $ p $).







        share|cite|improve this answer














        share|cite|improve this answer



        share|cite|improve this answer








        edited Dec 4 '18 at 20:20

























        answered Dec 4 '18 at 20:14









        StarfallStarfall

        13.3k11139




        13.3k11139






























            draft saved

            draft discarded




















































            Thanks for contributing an answer to Mathematics Stack Exchange!


            • Please be sure to answer the question. Provide details and share your research!

            But avoid



            • Asking for help, clarification, or responding to other answers.

            • Making statements based on opinion; back them up with references or personal experience.


            Use MathJax to format equations. MathJax reference.


            To learn more, see our tips on writing great answers.




            draft saved


            draft discarded














            StackExchange.ready(
            function () {
            StackExchange.openid.initPostLogin('.new-post-login', 'https%3a%2f%2fmath.stackexchange.com%2fquestions%2f3025063%2flet-k-f-2x-and-q-tx-x2-x-t-in-kx-show-that-q-t-is-not-solvabl%23new-answer', 'question_page');
            }
            );

            Post as a guest















            Required, but never shown





















































            Required, but never shown














            Required, but never shown












            Required, but never shown







            Required, but never shown

































            Required, but never shown














            Required, but never shown












            Required, but never shown







            Required, but never shown







            Popular posts from this blog

            Plaza Victoria

            In PowerPoint, is there a keyboard shortcut for bulleted / numbered list?

            How to put 3 figures in Latex with 2 figures side by side and 1 below these side by side images but in...